LSAT and Law School Admissions Forum

Get expert LSAT preparation and law school admissions advice from PowerScore Test Preparation.

 rahimlsat
  • Posts: 8
  • Joined: Oct 24, 2018
|
#59856
Could you please clarify why B) is wrong?

Initially I thought that since this is a "could be true" question, the 4 incorrect answer choices CANNOT be true. I immediately dismissed A) because of the not-law I had for Y under the first spoon. For B) though, I made a quick diagram:

1: W T Z
2: Y
3: U X

Seeing that this conforms to all the rules, I thought okay Z is in the first spoon, so this is the answer and I didn't even bother checking the others.

But when I checked the answers, it was D). I made another diagram of the only other place Y can be alone:

1: W T Z
2: U X
3: Y

This still conforms to all the rules. I realized that Z HAS to be in the first spoon, it has no choice. X on the other hand, can be either in the second or third.

So, for a "could be true" question, is a possible incorrect answer one that HAS to be true?
User avatar
 Dave Killoran
PowerScore Staff
  • PowerScore Staff
  • Posts: 5853
  • Joined: Mar 25, 2011
|
#59863
rahimlsat wrote:Could you please clarify why B) is wrong?

Initially I thought that since this is a "could be true" question, the 4 incorrect answer choices CANNOT be true. I immediately dismissed A) because of the not-law I had for Y under the first spoon. For B) though, I made a quick diagram:

1: W T Z
2: Y
3: U X

Seeing that this conforms to all the rules, I thought okay Z is in the first spoon, so this is the answer and I didn't even bother checking the others.

But when I checked the answers, it was D). I made another diagram of the only other place Y can be alone:

1: W T Z
2: U X
3: Y

This still conforms to all the rules. I realized that Z HAS to be in the first spoon, it has no choice. X on the other hand, can be either in the second or third.

So, for a "could be true" question, is a possible incorrect answer one that HAS to be true?
Hi Rahim,

Thanks for the question! I think you may have missed the last rule, which states that "The U is in the same spoonful as either the Y or the Z, but not both," because neither of your hypotheticals address that rule. You also mention Z being in the first spoonful, but due to that last rule Z must be with U in this question, and therefore Z cannot be in the first spoonful (and thus answer choice (B) is incorrect). If you rework the problem with that rule in mind, you'll see there are several solutions possible.

In answer to your specific question at the end, No, in a Could question a Must answer would actually be the correct answer, not an incorrect answer.


Please let me know if that helps. Thanks!

Get the most out of your LSAT Prep Plus subscription.

Analyze and track your performance with our Testing and Analytics Package.